Why doesn't L'hopitals Rule work for $limlimits_{x to infty} dfrac{x+ sin x}{x+ 2 sin x}$ [duplicate]












9












$begingroup$



This question already has an answer here:




  • Why doesn't L'Hopital's rule work in this case?

    4 answers




This is how I would evaluate $limlimits_{x to infty} dfrac{x+ sin x}{x+ 2 sin x}$



$=limlimits_{x to infty} dfrac{x left( 1+ frac{sin x}{x} right)}{x left(1+ 2 cdot frac{ sin x}{x} right)}$



$= dfrac{1+0}{1+2 cdot 0} = 1$



But now applying L'hopitals Rule, I get



$limlimits_{x to infty} dfrac{1+ cos x}{1+ 2 cos x}$



Since $cos x $ just oscillates between $[-1,1]$ I think we can conclude the limit doesn't exist.



What is going on here?










share|cite|improve this question









$endgroup$



marked as duplicate by Zacky, Namaste, Pierre-Guy Plamondon, Eric Wofsey, Lord Shark the Unknown Dec 28 '18 at 2:47


This question has been asked before and already has an answer. If those answers do not fully address your question, please ask a new question.


















  • $begingroup$
    See also here: math.stackexchange.com/q/1342202/515527 . This is a question raised quite frequently.
    $endgroup$
    – Zacky
    Dec 27 '18 at 13:50












  • $begingroup$
    This is also explained on the wikipedia page (en.wikipedia.org/wiki/…) where there is a simpler example such as $(x + cos x)/x$.
    $endgroup$
    – Ben
    Dec 27 '18 at 13:51












  • $begingroup$
    See also math.stackexchange.com/questions/1710786/…
    $endgroup$
    – Barry Cipra
    Dec 27 '18 at 14:06
















9












$begingroup$



This question already has an answer here:




  • Why doesn't L'Hopital's rule work in this case?

    4 answers




This is how I would evaluate $limlimits_{x to infty} dfrac{x+ sin x}{x+ 2 sin x}$



$=limlimits_{x to infty} dfrac{x left( 1+ frac{sin x}{x} right)}{x left(1+ 2 cdot frac{ sin x}{x} right)}$



$= dfrac{1+0}{1+2 cdot 0} = 1$



But now applying L'hopitals Rule, I get



$limlimits_{x to infty} dfrac{1+ cos x}{1+ 2 cos x}$



Since $cos x $ just oscillates between $[-1,1]$ I think we can conclude the limit doesn't exist.



What is going on here?










share|cite|improve this question









$endgroup$



marked as duplicate by Zacky, Namaste, Pierre-Guy Plamondon, Eric Wofsey, Lord Shark the Unknown Dec 28 '18 at 2:47


This question has been asked before and already has an answer. If those answers do not fully address your question, please ask a new question.


















  • $begingroup$
    See also here: math.stackexchange.com/q/1342202/515527 . This is a question raised quite frequently.
    $endgroup$
    – Zacky
    Dec 27 '18 at 13:50












  • $begingroup$
    This is also explained on the wikipedia page (en.wikipedia.org/wiki/…) where there is a simpler example such as $(x + cos x)/x$.
    $endgroup$
    – Ben
    Dec 27 '18 at 13:51












  • $begingroup$
    See also math.stackexchange.com/questions/1710786/…
    $endgroup$
    – Barry Cipra
    Dec 27 '18 at 14:06














9












9








9


2



$begingroup$



This question already has an answer here:




  • Why doesn't L'Hopital's rule work in this case?

    4 answers




This is how I would evaluate $limlimits_{x to infty} dfrac{x+ sin x}{x+ 2 sin x}$



$=limlimits_{x to infty} dfrac{x left( 1+ frac{sin x}{x} right)}{x left(1+ 2 cdot frac{ sin x}{x} right)}$



$= dfrac{1+0}{1+2 cdot 0} = 1$



But now applying L'hopitals Rule, I get



$limlimits_{x to infty} dfrac{1+ cos x}{1+ 2 cos x}$



Since $cos x $ just oscillates between $[-1,1]$ I think we can conclude the limit doesn't exist.



What is going on here?










share|cite|improve this question









$endgroup$





This question already has an answer here:




  • Why doesn't L'Hopital's rule work in this case?

    4 answers




This is how I would evaluate $limlimits_{x to infty} dfrac{x+ sin x}{x+ 2 sin x}$



$=limlimits_{x to infty} dfrac{x left( 1+ frac{sin x}{x} right)}{x left(1+ 2 cdot frac{ sin x}{x} right)}$



$= dfrac{1+0}{1+2 cdot 0} = 1$



But now applying L'hopitals Rule, I get



$limlimits_{x to infty} dfrac{1+ cos x}{1+ 2 cos x}$



Since $cos x $ just oscillates between $[-1,1]$ I think we can conclude the limit doesn't exist.



What is going on here?





This question already has an answer here:




  • Why doesn't L'Hopital's rule work in this case?

    4 answers








limits






share|cite|improve this question













share|cite|improve this question











share|cite|improve this question




share|cite|improve this question










asked Dec 27 '18 at 13:44









William William

1,207414




1,207414




marked as duplicate by Zacky, Namaste, Pierre-Guy Plamondon, Eric Wofsey, Lord Shark the Unknown Dec 28 '18 at 2:47


This question has been asked before and already has an answer. If those answers do not fully address your question, please ask a new question.









marked as duplicate by Zacky, Namaste, Pierre-Guy Plamondon, Eric Wofsey, Lord Shark the Unknown Dec 28 '18 at 2:47


This question has been asked before and already has an answer. If those answers do not fully address your question, please ask a new question.














  • $begingroup$
    See also here: math.stackexchange.com/q/1342202/515527 . This is a question raised quite frequently.
    $endgroup$
    – Zacky
    Dec 27 '18 at 13:50












  • $begingroup$
    This is also explained on the wikipedia page (en.wikipedia.org/wiki/…) where there is a simpler example such as $(x + cos x)/x$.
    $endgroup$
    – Ben
    Dec 27 '18 at 13:51












  • $begingroup$
    See also math.stackexchange.com/questions/1710786/…
    $endgroup$
    – Barry Cipra
    Dec 27 '18 at 14:06


















  • $begingroup$
    See also here: math.stackexchange.com/q/1342202/515527 . This is a question raised quite frequently.
    $endgroup$
    – Zacky
    Dec 27 '18 at 13:50












  • $begingroup$
    This is also explained on the wikipedia page (en.wikipedia.org/wiki/…) where there is a simpler example such as $(x + cos x)/x$.
    $endgroup$
    – Ben
    Dec 27 '18 at 13:51












  • $begingroup$
    See also math.stackexchange.com/questions/1710786/…
    $endgroup$
    – Barry Cipra
    Dec 27 '18 at 14:06
















$begingroup$
See also here: math.stackexchange.com/q/1342202/515527 . This is a question raised quite frequently.
$endgroup$
– Zacky
Dec 27 '18 at 13:50






$begingroup$
See also here: math.stackexchange.com/q/1342202/515527 . This is a question raised quite frequently.
$endgroup$
– Zacky
Dec 27 '18 at 13:50














$begingroup$
This is also explained on the wikipedia page (en.wikipedia.org/wiki/…) where there is a simpler example such as $(x + cos x)/x$.
$endgroup$
– Ben
Dec 27 '18 at 13:51






$begingroup$
This is also explained on the wikipedia page (en.wikipedia.org/wiki/…) where there is a simpler example such as $(x + cos x)/x$.
$endgroup$
– Ben
Dec 27 '18 at 13:51














$begingroup$
See also math.stackexchange.com/questions/1710786/…
$endgroup$
– Barry Cipra
Dec 27 '18 at 14:06




$begingroup$
See also math.stackexchange.com/questions/1710786/…
$endgroup$
– Barry Cipra
Dec 27 '18 at 14:06










2 Answers
2






active

oldest

votes


















15












$begingroup$

L'Hospital's rule contains an assumption that $lim_{x to a} f'(x)/g'(x)$ exists, which is not true in this case.






share|cite|improve this answer









$endgroup$









  • 3




    $begingroup$
    Unfortunately, students often forget the hypotheses for a result when they try to apply it.
    $endgroup$
    – GEdgar
    Dec 27 '18 at 13:51



















2












$begingroup$

Because your function doesn't satisfy the hypothesis. If you are studing the limit $xto c$, in order to apply the theorem the function $g=x+2sin x$ must be differentiable and $g'(x)ne 0$ in an open interval containing $c$, except in $c$. That means that you need a set (M,+infty) where $g' ne 0$. But $g'(x)=0$ $forall x=-frac{pi}{4}+2kpi$, so it doesn't exists a set like that.






share|cite|improve this answer









$endgroup$













  • $begingroup$
    The explanation is true and thus +1, yet the OP's question seems to point towards the fact that he forgot that L'Hospital's Rule applies in one direction only. Observe that if the denominator was $;3x+2sin x;$ then L'Hospital is appliable...yet it still wouldn't help as the limit of the quotient of the derivatives doesn't exist ...
    $endgroup$
    – DonAntonio
    Dec 27 '18 at 14:06












  • $begingroup$
    Thank you for the explanation!
    $endgroup$
    – ecrin
    Dec 27 '18 at 14:09


















2 Answers
2






active

oldest

votes








2 Answers
2






active

oldest

votes









active

oldest

votes






active

oldest

votes









15












$begingroup$

L'Hospital's rule contains an assumption that $lim_{x to a} f'(x)/g'(x)$ exists, which is not true in this case.






share|cite|improve this answer









$endgroup$









  • 3




    $begingroup$
    Unfortunately, students often forget the hypotheses for a result when they try to apply it.
    $endgroup$
    – GEdgar
    Dec 27 '18 at 13:51
















15












$begingroup$

L'Hospital's rule contains an assumption that $lim_{x to a} f'(x)/g'(x)$ exists, which is not true in this case.






share|cite|improve this answer









$endgroup$









  • 3




    $begingroup$
    Unfortunately, students often forget the hypotheses for a result when they try to apply it.
    $endgroup$
    – GEdgar
    Dec 27 '18 at 13:51














15












15








15





$begingroup$

L'Hospital's rule contains an assumption that $lim_{x to a} f'(x)/g'(x)$ exists, which is not true in this case.






share|cite|improve this answer









$endgroup$



L'Hospital's rule contains an assumption that $lim_{x to a} f'(x)/g'(x)$ exists, which is not true in this case.







share|cite|improve this answer












share|cite|improve this answer



share|cite|improve this answer










answered Dec 27 '18 at 13:48









littleOlittleO

30k647110




30k647110








  • 3




    $begingroup$
    Unfortunately, students often forget the hypotheses for a result when they try to apply it.
    $endgroup$
    – GEdgar
    Dec 27 '18 at 13:51














  • 3




    $begingroup$
    Unfortunately, students often forget the hypotheses for a result when they try to apply it.
    $endgroup$
    – GEdgar
    Dec 27 '18 at 13:51








3




3




$begingroup$
Unfortunately, students often forget the hypotheses for a result when they try to apply it.
$endgroup$
– GEdgar
Dec 27 '18 at 13:51




$begingroup$
Unfortunately, students often forget the hypotheses for a result when they try to apply it.
$endgroup$
– GEdgar
Dec 27 '18 at 13:51











2












$begingroup$

Because your function doesn't satisfy the hypothesis. If you are studing the limit $xto c$, in order to apply the theorem the function $g=x+2sin x$ must be differentiable and $g'(x)ne 0$ in an open interval containing $c$, except in $c$. That means that you need a set (M,+infty) where $g' ne 0$. But $g'(x)=0$ $forall x=-frac{pi}{4}+2kpi$, so it doesn't exists a set like that.






share|cite|improve this answer









$endgroup$













  • $begingroup$
    The explanation is true and thus +1, yet the OP's question seems to point towards the fact that he forgot that L'Hospital's Rule applies in one direction only. Observe that if the denominator was $;3x+2sin x;$ then L'Hospital is appliable...yet it still wouldn't help as the limit of the quotient of the derivatives doesn't exist ...
    $endgroup$
    – DonAntonio
    Dec 27 '18 at 14:06












  • $begingroup$
    Thank you for the explanation!
    $endgroup$
    – ecrin
    Dec 27 '18 at 14:09
















2












$begingroup$

Because your function doesn't satisfy the hypothesis. If you are studing the limit $xto c$, in order to apply the theorem the function $g=x+2sin x$ must be differentiable and $g'(x)ne 0$ in an open interval containing $c$, except in $c$. That means that you need a set (M,+infty) where $g' ne 0$. But $g'(x)=0$ $forall x=-frac{pi}{4}+2kpi$, so it doesn't exists a set like that.






share|cite|improve this answer









$endgroup$













  • $begingroup$
    The explanation is true and thus +1, yet the OP's question seems to point towards the fact that he forgot that L'Hospital's Rule applies in one direction only. Observe that if the denominator was $;3x+2sin x;$ then L'Hospital is appliable...yet it still wouldn't help as the limit of the quotient of the derivatives doesn't exist ...
    $endgroup$
    – DonAntonio
    Dec 27 '18 at 14:06












  • $begingroup$
    Thank you for the explanation!
    $endgroup$
    – ecrin
    Dec 27 '18 at 14:09














2












2








2





$begingroup$

Because your function doesn't satisfy the hypothesis. If you are studing the limit $xto c$, in order to apply the theorem the function $g=x+2sin x$ must be differentiable and $g'(x)ne 0$ in an open interval containing $c$, except in $c$. That means that you need a set (M,+infty) where $g' ne 0$. But $g'(x)=0$ $forall x=-frac{pi}{4}+2kpi$, so it doesn't exists a set like that.






share|cite|improve this answer









$endgroup$



Because your function doesn't satisfy the hypothesis. If you are studing the limit $xto c$, in order to apply the theorem the function $g=x+2sin x$ must be differentiable and $g'(x)ne 0$ in an open interval containing $c$, except in $c$. That means that you need a set (M,+infty) where $g' ne 0$. But $g'(x)=0$ $forall x=-frac{pi}{4}+2kpi$, so it doesn't exists a set like that.







share|cite|improve this answer












share|cite|improve this answer



share|cite|improve this answer










answered Dec 27 '18 at 14:02









ecrinecrin

3477




3477












  • $begingroup$
    The explanation is true and thus +1, yet the OP's question seems to point towards the fact that he forgot that L'Hospital's Rule applies in one direction only. Observe that if the denominator was $;3x+2sin x;$ then L'Hospital is appliable...yet it still wouldn't help as the limit of the quotient of the derivatives doesn't exist ...
    $endgroup$
    – DonAntonio
    Dec 27 '18 at 14:06












  • $begingroup$
    Thank you for the explanation!
    $endgroup$
    – ecrin
    Dec 27 '18 at 14:09


















  • $begingroup$
    The explanation is true and thus +1, yet the OP's question seems to point towards the fact that he forgot that L'Hospital's Rule applies in one direction only. Observe that if the denominator was $;3x+2sin x;$ then L'Hospital is appliable...yet it still wouldn't help as the limit of the quotient of the derivatives doesn't exist ...
    $endgroup$
    – DonAntonio
    Dec 27 '18 at 14:06












  • $begingroup$
    Thank you for the explanation!
    $endgroup$
    – ecrin
    Dec 27 '18 at 14:09
















$begingroup$
The explanation is true and thus +1, yet the OP's question seems to point towards the fact that he forgot that L'Hospital's Rule applies in one direction only. Observe that if the denominator was $;3x+2sin x;$ then L'Hospital is appliable...yet it still wouldn't help as the limit of the quotient of the derivatives doesn't exist ...
$endgroup$
– DonAntonio
Dec 27 '18 at 14:06






$begingroup$
The explanation is true and thus +1, yet the OP's question seems to point towards the fact that he forgot that L'Hospital's Rule applies in one direction only. Observe that if the denominator was $;3x+2sin x;$ then L'Hospital is appliable...yet it still wouldn't help as the limit of the quotient of the derivatives doesn't exist ...
$endgroup$
– DonAntonio
Dec 27 '18 at 14:06














$begingroup$
Thank you for the explanation!
$endgroup$
– ecrin
Dec 27 '18 at 14:09




$begingroup$
Thank you for the explanation!
$endgroup$
– ecrin
Dec 27 '18 at 14:09



Popular posts from this blog

To store a contact into the json file from server.js file using a class in NodeJS

Redirect URL with Chrome Remote Debugging Android Devices

Dieringhausen